Prove A is Diagonalizable (Actual Question)

  • Thread starter Thread starter pezola
  • Start date Start date
Click For Summary
A matrix A with two distinct eigenvalues, λ1 and λ2, and a dimension of eigenspace for λ1 equal to n-1 is diagonalizable. The dimension of the eigenspace for λ2 must be at least 1, leading to the conclusion that it is exactly 1. This means the total dimension of the eigenspaces equals n, satisfying the requirement for diagonalizability. The eigenvectors corresponding to these eigenvalues form a basis for the space, confirming A is diagonalizable. Understanding the independence of these eigenvectors is crucial in the proof.
pezola
Messages
11
Reaction score
0
[SOLVED] Prove A is Diagonalizable (Actual Question)

Homework Statement




Suppose that A \in M^{nxn}(F) and has two distinct eigenvalues, \lambda_{1} and \lambda_{2}, and that dim(E(subscript \lambda_{1} ))= n-1. Prove that A is diagonalizable.


The Attempt at a Solution



So far, I know that dim(E subscript \lambda2) \geq1
and that
dim(E subscript \lambda1) + dim(E subscript \lambda2) \leq n.
So dim(E subscript \lambda2) = 1.

I am not exactly sure how this helps me to show A is digonalizable. Maybe I am thinking of something else and don't need this to prove A is diagonalizable. Please help.

(Also, sorry about my prevous blank post; I am new)
 
Physics news on Phys.org
Welcome to PF!

pezola said:
So far, I know that dim(E subscript \lambda2) \geq1
and that
dim(E subscript \lambda1) + dim(E subscript \lambda2) \leq n.
So dim(E subscript \lambda2) = 1.

Hi pezola! Welcome to PF! :smile:

Your reasoning seems fine. :smile:

Can't you now use proof by induction - that is, assume the theorem is true for all numbers up to n - 1, and then prove it for n?
(Also, sorry about my prevous blank post; I am new)

:smile: … no problemo! … :smile:
 
A matrix is diagonalizable iff its eigenvectors form a basis for the space (do you understand why?). You've pretty much shown this, maybe just another word or too on why the eigenvectors are independent.
 
Question: A clock's minute hand has length 4 and its hour hand has length 3. What is the distance between the tips at the moment when it is increasing most rapidly?(Putnam Exam Question) Answer: Making assumption that both the hands moves at constant angular velocities, the answer is ## \sqrt{7} .## But don't you think this assumption is somewhat doubtful and wrong?

Similar threads

  • · Replies 2 ·
Replies
2
Views
2K
Replies
3
Views
3K
  • · Replies 29 ·
Replies
29
Views
3K
  • · Replies 8 ·
Replies
8
Views
3K
Replies
7
Views
2K
Replies
3
Views
1K
  • · Replies 3 ·
Replies
3
Views
4K
  • · Replies 5 ·
Replies
5
Views
1K
Replies
1
Views
2K
  • · Replies 1 ·
Replies
1
Views
4K